Difference between revisions of "2019 AMC 10B Problems/Problem 13"

Line 5: Line 5:
 
<math>\textbf{(A) } -5 \qquad\textbf{(B) } 0 \qquad\textbf{(C) } 5 \qquad\textbf{(D) } \frac{15}{4} \qquad\textbf{(E) } \frac{35}{4}</math>
 
<math>\textbf{(A) } -5 \qquad\textbf{(B) } 0 \qquad\textbf{(C) } 5 \qquad\textbf{(D) } \frac{15}{4} \qquad\textbf{(E) } \frac{35}{4}</math>
  
 +
==Solution==
 +
There are <math>3</math> cases: <math>6</math> is the median, <math>8</math> is the median, and <math>x</math> is the median. In all cases, the mean is <math>7+\frac{x}{5}</math>.<br>
 +
For case 1, <math>x=-5</math>. This allows 6 to be the median because the set is <math>-5,4,6,8,17</math>.<br>
 +
For case 2, <math>x=5</math>. This is an extraneous case because the set is <math>4,5,6,8,17</math>.<br>
 +
For case 3, <math>x=\frac{35}{4}</math>. This is an extraneous case because the set is <math>4,6,8,\frac{35}{4},17</math>.<br>
 +
Only case 1 yields a solution, <math>x=-5</math>, so the answer is <math>\textbf{(A) } -5</math>.
 
==See Also==
 
==See Also==
  
 
{{AMC10 box|year=2019|ab=B|num-b=12|num-a=14}}
 
{{AMC10 box|year=2019|ab=B|num-b=12|num-a=14}}
 
{{MAA Notice}}
 
{{MAA Notice}}

Revision as of 13:16, 14 February 2019

Problem

What is the sum of all real numbers $x$ for which the median of the numbers $4,6,8,17,$ and $x$ is equal to the mean of those five numbers?

$\textbf{(A) } -5 \qquad\textbf{(B) } 0 \qquad\textbf{(C) } 5 \qquad\textbf{(D) } \frac{15}{4} \qquad\textbf{(E) } \frac{35}{4}$

Solution

There are $3$ cases: $6$ is the median, $8$ is the median, and $x$ is the median. In all cases, the mean is $7+\frac{x}{5}$.
For case 1, $x=-5$. This allows 6 to be the median because the set is $-5,4,6,8,17$.
For case 2, $x=5$. This is an extraneous case because the set is $4,5,6,8,17$.
For case 3, $x=\frac{35}{4}$. This is an extraneous case because the set is $4,6,8,\frac{35}{4},17$.
Only case 1 yields a solution, $x=-5$, so the answer is $\textbf{(A) } -5$.

See Also

2019 AMC 10B (ProblemsAnswer KeyResources)
Preceded by
Problem 12
Followed by
Problem 14
1 2 3 4 5 6 7 8 9 10 11 12 13 14 15 16 17 18 19 20 21 22 23 24 25
All AMC 10 Problems and Solutions

The problems on this page are copyrighted by the Mathematical Association of America's American Mathematics Competitions. AMC logo.png